Tricky Maxima..!!!

Algebra Level 4

Over all sets of real numbers ( x , y ) (x,y) such that x 2 + y 2 = 1 { x }^{ 2 }+{ y }^{ 2 }=1 , find the maximum value of F ( x , y ) = ( 3 x + 4 y ) 2 + ( 8 x + 6 y ) 2 F(x,y)= ({ 3x+4y })^{ 2 } + { (8x+6y) }^{ 2 }

to 2 decimal places

This is part of this set


The answer is 123.41.

This section requires Javascript.
You are seeing this because something didn't load right. We suggest you, (a) try refreshing the page, (b) enabling javascript if it is disabled on your browser and, finally, (c) loading the non-javascript version of this page . We're sorry about the hassle.

5 solutions

Since x 2 + y 2 = 1 x^2+y^2 = 1 , we can let x = cos θ x = \cos \theta and y = sin θ y = \sin \theta . Therefore, we have:

F ( x , y ) = ( 3 x + 4 y ) 2 + ( 8 x + 6 y ) 2 = 73 x 2 + 120 x y + 52 y 2 As x 2 + y 2 = 1 = 21 x 2 + 120 x y + 52 = 21 cos 2 θ + 120 cos θ sin θ + 52 = 21 2 ( cos ( 2 θ ) + 1 ) + 60 sin ( 2 θ ) + 52 = 10.5 cos ( 2 θ ) + 60 sin ( 2 θ ) + 62.5 = 10. 5 2 + 6 0 2 sin ( 2 θ + ϕ ) + 62.5 where ϕ = tan 1 10.5 60 \begin{aligned} F(x,y) & = (3x+4y)^2 + (8x + 6y)^2 \\ & = 73x^2 +120xy +52y^2 \quad \quad \small \color{#3D99F6}{\text{As } x^2+y^2 = 1} \\ & = 21x^2 +120xy +52 \\ & = 21\cos^2 \theta +120\cos \theta \sin \theta +52 \\ & = \frac{21}{2}(\cos (2 \theta) +1 ) +60 \sin (2 \theta) +52 \\ & = 10.5 \cos (2 \theta) + 60 \sin (2 \theta)+62.5 \\ & = \sqrt{10.5^2 +60^2} \sin (2 \theta + \color{#3D99F6}{\phi})+62.5 \quad \quad \small \color{#3D99F6}{\text{where } \phi = \tan^{-1} \frac{10.5}{60}} \end{aligned}

Since maximum of sin ( 2 θ + ϕ ) = 1 \sin (2 \theta +\phi) = 1 , then we have:

F ( x , y ) m a x = 10. 5 2 + 6 0 2 + 62.5 = 60.91182151 + 62.5 123.41 \begin{aligned} F(x,y)_{max} & = \sqrt{10.5^2 +60^2}+62.5 \\ & = 60.91182151 + 62.5 \\ & \approx \boxed{123.41} \end{aligned}

Nice approach.(+1.

Niranjan Khanderia - 2 years, 11 months ago
Abhishek Sinha
Aug 19, 2014

Consider the linear transformation A = ( 3 4 8 6 ) A=\begin{pmatrix} 3 & 4\\ 8& 6\end{pmatrix} . By performing Singular Value Decomposition of this matrix, we find that the maximum singular value of the matrix A A is 11.1091 11.1091 . Thus the maximum value of F ( x , y ) F(x,y) is simply the Rayleigh quotient of A T A A^{T}A , which is given by the square of the maximum singular value of A A , i.e., 11.109 1 2 123.41 11.1091^2\approx 123.41 .

Ankit Akash
Sep 5, 2014

We substitute x=sinß to get y=cosß from 1st relation. Now we write see second equation in form of sin2ß and cos 2ß and apply Cauchy Schwartz to get the answer

did the same way!

Mvs Saketh - 6 years, 3 months ago

exactly what i did

Mayank Singh - 6 years, 2 months ago
Hansraj Sharma
Aug 18, 2014

here is a tricky method first we take x=0,y=1 then F(x,y)=16+36=52 second we take x=1,y=0 then F(x,y)=9+64=73 after that 52+73=125........got it

Moderator note:

This solution is incorrect. There is no justification, nor is it correct, that max F ( x , y ) = F ( 0 , 1 ) + F ( 1 , 0 ) \max F(x,y) = F(0,1) + F(1,0) .

Why did u added the two values ???? That will give value of 2f(x,y)...

Sandeep Bhardwaj - 6 years, 9 months ago

The elegant solution is :: as give x^{2} +y^{2} = 1 ..we can assume x=sin(a) and y=cos(a)..so max value of 3sin(a) + 4cos(a) = 5. and max value of 8sin(a) + 6cos(a) =10. Hence max value of f(x,y)=5^2 + 10^2 = 125.

Sandeep Bhardwaj - 6 years, 9 months ago

Log in to reply

This is not justified as the two expressions attain maximum values at different values of a a , namely a = 2 tan 1 1 3 a=2\tan^{-1} {\dfrac{1}{3}} and a = 2 tan 1 1 2 a=2\tan^{-1} {\dfrac{1}{2}} .

The correct approach would be to maximize 21 2 ( cos 2 a sin 2 a ) + 120 sin a cos a + 125 2 = 1 2 ( 120 sin 2 a + 21 cos 2 a + 125 ) \dfrac{21}{2} (\cos^2 a-\sin^2 a) +120\sin a \cos a+\dfrac{125}{2}\\ =\dfrac{1}{2} (120\sin 2a+21\cos 2a+125)

Hence, the maximum value of the expression would be max { F ( x , y ) } = 12 0 2 + 2 1 2 + 125 2 = 3 1649 + 125 2 123.4118 \begin{aligned} \max\{F(x,y)\} &=\dfrac{\sqrt{120^2+21^2}+125}{2}\\ &=\dfrac{3\sqrt {1649}+125}{2}\\ &\approx \boxed{123.4118} \end{aligned}

So the answer provided is wrong.

Pratik Shastri - 6 years, 9 months ago

Log in to reply

I too have done the same thing and got the same answer

Ronak Agarwal - 6 years, 9 months ago

first i find 123.412 but that was wrong so i take a tricky formula. it is just tricky not real.

hansraj sharma - 6 years, 9 months ago

L e t F ( x , y ) = ( 3 x + 4 y ) 2 + 4 ( 4 x + 3 y ) 2 L e t F ( m , n ) b e t h e p o i n t w h e r e F i s m a x i m u m , w i t h c o n d i t i o n t h a t i t i s a l s o o n x 2 + y 2 = 1. t h e y h a v e c o m m o n t a n g e n t a t ( m , n ) . d y d x m , n f r o m F ( x , y ) = 0 a n d x 2 + y 2 = 1 , a r e e q u a l . 2 x + 2 y d y d x = 0. d y d x = x y . A n d d F d x = 2 ( 3 x + 4 y ) ( 3 + 4 d y d x ) + 8 ( 4 x + 3 y ) ( 4 + 3 d y d x ) = 0. B u t d y d x = x y , s u b s t i t u t i n g d y d x b y x y , a n d m u l t i p l y i n g b y y 2 , d F d x = ( 3 x + 4 y ) ( 3 y 4 x ) + 4 ( 4 x + 3 y ) ( 4 y 3 x ) = 0. 12 x 2 7 x y + 12 y 2 48 x 2 + 28 x y + 48 y 2 = 60 x 2 + 21 x y + 60 y 2 = 0. 20 ( y x ) 2 + 7 ( y x ) 20 = 0 S o l v i n g q u a d r a t i c i n y x , y x = 0.840197............. ( A ) y 2 x 2 = 0.705931 1 . y 2 x 2 + y 2 = y 2 1 = 0.705931 1.705931 . y = 0.705931 1.705931 = 0.64328. b y ( A ) x = 0.64328 0.840197 = 0.76563. S o ( m , n ) = ( 0.76563 , 0.64328 ) . F ( m , n ) = 123.437. Let~~F(x,y)=(3x+4y)^2+4*(4x+3y)^2\\ ~~~~\\ Let~F(m,n)~be~the~~point~where~F~is~maximum,~with ~condition~that~ it~ is~also~on~x^2+y^2=1.\\ \therefore~~ they~ have~ common~ tangent ~at~(m,n).~~~\implies~~\dfrac{dy}{dx}_{m,n}~from~F(x,y)=0~~and~~x^2+y^2=1, ~are~equal.\\ ~~~\\ 2x+2y\dfrac{dy}{dx}=0.~~\implies~\dfrac{dy}{dx}=-\dfrac x y.\\ And~~\dfrac{dF}{dx}=2(3x+4y)*(3+4\dfrac{dy}{dx})+ 8*(4x+3y)*(4+3\dfrac{dy}{dx})=0.\\ ~~~~\\ ~~~\\ But~\dfrac{dy}{dx}=-\dfrac x y,~~substituting~\dfrac{dy}{dx}~~by~~-\dfrac x y,~~and ~multiplying~by~\dfrac y 2,\\ \dfrac{dF}{dx}= (3x+4y)*(3y-4x)+ 4*(4x+3y)*(4y-3x)=0.\\ \therefore~-12x^2-7xy+12y^2-48x^2+28xy+48y^2= -60x^2+21xy+60y^2=0.\\ \implies~~20(\dfrac y x)^2+7(\dfrac y x)-20=0\\ Solving~quadratic~in~\dfrac y x,~~~~~\dfrac y x =0.840197.............(A)\\ ~~~~~~\\ ~~~\\ \therefore~~~~\dfrac {y^2}{ x^2}=\dfrac{0.705931} 1.\\ \implies~ \dfrac {y^2}{ x^2+y^2}=\dfrac {y^2} 1 = \dfrac {0.705931}{ 1.705931}.\\ \therefore~~~y=\sqrt{\dfrac {0.705931}{ 1.705931} }=0.64328.\\ \therefore~by~(A)~~x=\dfrac{0.64328}{0.840197}=0.76563.\\ ~~~\\ ~~~~\\ So~~(m,n)=(0.76563,0.64328).\\ \therefore~~F(m,n)=\Huge~~\color{#D61F06}{123.437}.

L e t F ( x , y ) = ( 3 x + 4 y ) 2 + 4 ( 4 x + 3 y ) 2 L e t F ( m , n ) b e t h e p o i n t w h e r e F i s m a x i m u m , w i t h c o n d i t i o n t h a t i t i s a l s o o n x 2 + y 2 = 1. t h e y h a v e c o m m o n t a n g e n t a t ( m , n ) . d y d x m , n f r o m F ( x , y ) = 0 a n d x 2 + y 2 = 1 , a r e e q u a l . 2 x + 2 y d y d x = 0. d y d x = x y . A n d d F d x = 2 ( 3 x + 4 y ) ( 3 + 4 d y d x ) + 8 ( 4 x + 3 y ) ( 4 + 3 d y d x ) = 0. B u t d y d x = x y , s u b s t i t u t i n g d y d x b y x y , a n d m u l t i p l y i n g b y y 2 , d F d x = ( 3 x + 4 y ) ( 3 y 4 x ) + 4 ( 4 x + 3 y ) ( 4 y 3 x ) = 0. 12 x 2 7 x y + 12 y 2 48 x 2 + 28 x y + 48 y 2 = 60 x 2 + 21 x y + 60 y 2 = 0. 20 ( y x ) 2 + 7 ( y x ) 20 = 0 S o l v i n g q u a d r a t i c i n y x , y x = 0.840197............. ( A ) y 2 x 2 = 0.705931 1 . y 2 x 2 + y 2 = y 2 1 = 0.705931 1.705931 . y = 0.705931 1.705931 = 0.64328. b y ( A ) x = 0.64328 0.840197 = 0.76563. S o ( m , n ) = ( 0.76563 , 0.64328 ) . F ( m , n ) = 123.437. Let~~F(x,y)=(3x+4y)^2+4*(4x+3y)^2\\ ~~~~\\ Let~F(m,n)~be~the~~point~where~F~is~maximum,~with ~condition~that~ it~ is~also~on~x^2+y^2=1.\\ \therefore~~ they~ have~ common~ tangent ~at~(m,n).~~~\implies~~\dfrac{dy}{dx}_{m,n}~from~F(x,y)=0~~and~~x^2+y^2=1, ~are~equal.\\ ~~~~\\ ~~~\\ 2x+2y\dfrac{dy}{dx}=0.~~\implies~\dfrac{dy}{dx}=-\dfrac x y.\\ And~~\dfrac{dF}{dx}=2(3x+4y)*(3+4\dfrac{dy}{dx})+ 8*(4x+3y)*(4+3\dfrac{dy}{dx})=0.\\ ~~~~\\ ~~~\\ But~\dfrac{dy}{dx}=-\dfrac x y,~~substituting~\dfrac{dy}{dx}~~by~~-\dfrac x y,~~and ~multiplying~by~\dfrac y 2,\\ \dfrac{dF}{dx}= (3x+4y)*(3y-4x)+ 4*(4x+3y)*(4y-3x)=0.\\ \therefore~-12x^2-7xy+12y^2-48x^2+28xy+48y^2= -60x^2+21xy+60y^2=0.\\ \implies~~20(\dfrac y x)^2+7(\dfrac y x)-20=0\\ Solving~quadratic~in~\dfrac y x,~~~~~\dfrac y x =0.840197.............(A)\\ ~~~~~~\\ ~~~\\ \therefore~~~~\dfrac {y^2}{ x^2}=\dfrac{0.705931} 1.\\ \implies~ \dfrac {y^2}{ x^2+y^2}=\dfrac {y^2} 1 = \dfrac {0.705931}{ 1.705931}.\\ \therefore~~~y=\sqrt{\dfrac {0.705931}{ 1.705931} }=0.64328.\\ \therefore~by~(A)~~x=\dfrac{0.64328}{0.840197}=0.76563.\\ ~~~\\ ~~~~\\ So~~(m,n)=(0.76563,0.64328).\\ \therefore~~F(m,n)=\Huge~~\color{#D61F06}{123.437}.

Niranjan Khanderia - 2 years ago

0 pending reports

×

Problem Loading...

Note Loading...

Set Loading...